Chapter 4 Practice Test 1: Answers and explanations - Part II Practice Test 1

PSAT 8/9 Prep with 2 practice tests - Princeton Review 2020

Chapter 4 Practice Test 1: Answers and explanations
Part II Practice Test 1

PRACTICE TEST 1 ANSWER KEY

PRACTICE TEST 1 EXPLANATIONS

Section 1—Reading

1.A

The question asks about a main theme of the passage. Since this is a general question, it should be answered after the specific questions. The passage is a story about the narrator and his friend, Hassan. The narrator states in the second paragraph, Over the years, I had seen a lot of guys run kites. But Hassan was by far the greatest kite runner I’d ever seen, and the majority of the passage gives a specific example of Hassan’s expertise during a kite-fighting tournament. The narrator follows Hassan, even though it appears that Hassan is running the wrong way to catch the kite. At the end of the passage, Hassan catches the kite, which shows that the narrator was right to trust Hassan. Eliminate answers that don’t match the prediction. Keep (A) as it matches the prediction. Eliminate (B) because the passage doesn’t focus on hard work; it focuses on Hassan’s talent and on the relationship between Hassan and the narrator. Eliminate (C) because only Hassan is successful in catching the kite; the passage does not mention the other competitors doing their best. Therefore, the passage does not focus on competition bringing out the best in each competitor. Choice (D) matches the narrator’s initial uncertainty, but since Hassan is successful, the passage indicates that the narrator was right to trust Hassan, rather than to be cautious. Eliminate (D). The correct answer is (A).

2.C

The question asks why the author includes the second paragraph. Read the second paragraph as the window. The paragraph states, But Hassan was by far the greatest kite runner I’d ever seen. It was downright eerie the way he always got to the spot the kite would land before the kite did, as if he had some sort of inner compass. This emphasizes what the narrator thought about his friend Hassan’s skill. Eliminate answers that don’t match the prediction. Eliminate (A) because, although the second paragraph indicates that Hassan is a skilled kite-runner, it does not mention how he became skilled. This is a Mostly Right, Slightly Wrong trap answer. Eliminate (B) because there is no mention of competitiveness between the characters. This is a Could Be True trap answer that is not supported by direct evidence from the passage. Keep (C) because it matches the prediction; the narrator states his impression that Hassan was a skilled kite-runner. Eliminate (D) because the paragraph only focuses on Hassan and the narrator; it does not describe any time period. The correct answer is (C).

3.B

The question asks for the evidence that best supports the narrator’s description of Hassan as the greatest kite runner I’d ever seen (lines 13—14). Read the lines for each answer, eliminating any that do not support this description. The lines for (A) state that the narrator had seen a lot of guys run kites, but they don’t mention Hassan, so eliminate (A). The lines for (B) refer to Hassan, saying, It was downright eerie the way he always got to the spot the kite would land before the kite did, as if he had some sort of inner compass. These lines support the description of Hassan as a great kite runner, so keep (B). The lines for (C) state that the narrator fell behind Hassan when they ran, but they don’t directly support the idea that Hassan is a great kite runner. Eliminate (C). The lines for (D) describe the approaching kite runners, but they don’t mention Hassan, so eliminate (D). The correct answer is (B).

4.A

The question asks what the phrase “inner compass” most strongly suggests about Hassan. Use the given line reference to find the window. The passage states It was downright eerie the way he always got to the spot the kite would land before the kite did, as if he had some sort of inner compass, meaning that Hassan is naturally very good at finding kites. Eliminate answers that don’t match the prediction. Keep (A) because it matches the prediction. Eliminate (B) because there is no indication that Hassan is in a strange city. In fact, the passage implies that the children know this city very well. This is a Mostly Right, Slightly Wrong trap answer. Choice (C) can be eliminated because, while Hassan seems to be guided in the direction of the kite by an inner compass, the passage does not mention morals. This is a Right Words, Wrong Meaning trap answer based on another meaning of inner compass that is not supported by the passage. Eliminate (D) because there is no evidence that Hassan is cheating. The correct answer is (A).

5.C

The question asks what the word ragged most nearly means in line 22. Go back to the text, find the word ragged, and cross it out. Then read the window carefully, using context clues to determine another word that would fit in the text. The text says, “Hassan! Wait!” I yelled, my breathing hot and ragged. The narrator is running after Hassan, who is faster, so the narrator is struggling to breathe normally. Therefore, ragged must mean something like “uneven.” Fast does not match “uneven,” so eliminate (A). This is a Could Be True trap answer; it is likely that the narrator’s breath is also fast, since he is running, but this is not the meaning of ragged. Loud does not match “uneven,” so eliminate (B). Rough matches “uneven,” so keep (C). Fevered does not match “uneven,” so eliminate (D). This is a Right Answer, Wrong Question trap answer; fevered could match hot, but it is not the meaning of ragged. The correct answer is (C).

6.B

The question asks what Hassan’s actions suggest about him after he and the narrator reach the middle school. Notice that this is the first question in a paired set, so it can be done in tandem with Q7. Look at the answer choices for Q7 first. The lines for (7A) come from the first paragraph, before the boys reach the middle school. Therefore, these lines do not answer Q6, so eliminate (7A). The lines for (7B) describe the narrator’s actions rather than Hassan’s actions. These lines do not answer Q6, so eliminate (7B). The lines for (7C) are spoken by the narrator; since these lines do not describe Hassan’s actions, they do not answer Q6. Eliminate (7C). The lines for (7D) say, Hassan popped a mulberry in his mouth. “It’s coming,” he said. Check to see whether these lines support any of the answers for Q6. They support (6B): Hassan shows his confidence by sitting calmly and eating while he waits for the kite, and by stating that the kite is coming. Draw a line connecting (7D) and (6B). Without any support in the answers from Q7, (6A), (6C), and (6D) can be eliminated. The correct answers are (6B) and (7D).

7.D

(See explanation above.)

8.C

The question asks what sound is heard while the kite is falling. Since there is no line reference, use lead words and the order of the questions to find the window for the question. The answers for questions 6 and 7 came from lines 50—51, so the window for Q8 is most likely after those lines. Beginning with line 52, look for information about the kite falling and a sound. Lines 55—57 state, I looked up, saw the kite plummeting toward us. I heard footfalls, shouts, an approaching melee of kite runners. The sounds being described come from the approaching group of competitors. Eliminate answers that don’t match the prediction. Eliminate (A) as the paragraph does not mention the wind. Eliminate (B): although Hassan calls out to the narrator earlier in the passage, he does not call out to the narrator while the kite is falling. This is a Right Answer, Wrong Question trap answer. Keep (C) as it matches the prediction. Choice (D) can be eliminated because the sound of parents cheering is mentioned in the first paragraph, but it does not occur while the kite is falling. This is another Right Answer, Wrong Question trap answer. The correct answer is (C).

9.D

The question asks about the main idea of the passage. Because this is a general question, it should be done after the specific questions. The first paragraph introduces Pokémon Go and says that the phenomenon has made its way through civilization and is now venturing into uncharted territory: national parks. The sixth paragraph captures the main idea of the passage, stating that Pokémon Go may become a new way to explore historic parks. Choice (A) is a Right Answer, Wrong Question trap answer; although the passage does mention that injuries abound when playing the game, this is not the main idea of the passage. Eliminate (A). Eliminate (B) because it does not mention Pokémon Go. Although (C) mentions both Pokémon Go and national parks, this is Right Words, Wrong Meaning trap answer. One of the park employees quoted in the passage says that games such as Pokémon Go are a new and emerging opportunity to bring new audiences to the park, but the passage doesn’t indicate that Pokémon Go players are more likely than others to visit national parks. Eliminate (C). Keep (D) because it matches the prediction. The correct answer is (D).

10.C

The question asks what the word create most nearly means in line 33. Go back to the text, find the word create, and cross it out. Then read the window carefully, using context clues to determine another word that would fit in the text. The text says, “One of our goals as part of the National Park Service Centennial is to connect with and create the next generation of park visitors, supporters, and advocates.” Therefore, create must mean something like “develop.” Choice (A) is a Could Be True trap answer; design can mean “to draw or plan out,” which is an alternate meaning of create that is not supported by the passage. Eliminate (A). Eliminate (B) because birth means “give life to,” which does not match “develop.” Keep (C) because foster matches “develop.” Help does not match “develop,” so eliminate (D). The correct answer is (C).

11.A

The question asks what the word emerging most nearly means in line 36. Go back to the text, find the word emerging, and cross it out. Then read the window carefully, using context clues to determine another word that would fit in the text. The text says, “Games that use geolocation are a new and emerging opportunity to bring new audiences to the park.” Therefore, emerging must mean something like “growing.” Developing matches “growing,” so keep (A). Appearing means “becoming visible,” which does not match “growing.” Eliminate (B). Becoming does not match “growing,” so eliminate (C). Note that (B) and (C) are Could Be True trap answers based on other meanings of emerging that are not supported by the text. Unexpected means “surprising,” which does not match “growing.” Eliminate (D). The correct answer is (A).

12.B

The question asks what assertion the example in lines 39—42 supports. This is the first question in a paired set, but it is easy to find, so it can be done on its own. Use the given line reference to find the window. The lines state, Great Smoky Mountains’s Mountain Farm Museum has three Pokéstops. When found, historical text will pop up on screen, and players can tap an icon to learn more before returning to the game. The previous sentence states that Pokémon Go also has an educational component to it. Therefore, the Mountain Farm Museum’s Pokéstops are examples of Pokémon Go’s educational components. Eliminate answers that don’t match the prediction. Eliminate (A) because there is no mention of injury in the window. Keep (B) because informing Pokémon Go players about the national parks matches the prediction. Eliminate (C) because there is no mention of the National Park Service Centennial in the window. Eliminate (D) because there is no indication that visitors demanded Pokémon Go. The correct answer is (B).

13.C

The question is the best evidence question in a paired set. Because Q12 was easy to find, simply look at the lines used to answer the previous question. Lines 38—42—Pokémon Go also has an educational component to it. Great Smoky Mountains’s Mountain Farm Museum has three Pokéstops. When found, historical text will pop up on screen, and players can tap an icon to learn more before returning to the game—were used to answer Q12. Of these lines, only lines 38—39 are given as an answer choice. The correct answer is (C).

14.A

The question asks what Lynda Doucette indicates. Notice that this is the first question in a paired set, so it can be done in tandem with Q15. Look at the answers for Q15 first. The lines of (15A) say that Lynda Doucette claims the park’s landscape obstructs cell service but that she and her colleagues have found at least five Pokéstops. Check the answers for Q14 to see whether any of the answers are supported by these lines. These lines do not support any of the answer choices in Q14; eliminate (15A). The lines in (15B) state, “It gets people out there,” Doucette says. “I think it’s an opportunity to bring a new audience to a site.” These lines do not support any of the answer choices in Q14; eliminate (15B). The lines in (15C) state, But as play increases, injuries abound. Already, players have been hurt after falling or walking into obstacles while cruising for critters. Although these lines may seem to match (14D), they were not spoken by Lynda Doucette, so they do not answer Q14. Eliminate (15C). The lines in (15D) state, “I don’t anticipate that we’re going to have any new rules implemented,” Doucette says. “It’s the same safety concerns we’ve had before this game.” These lines match (14A); since Doucette mentions the same safety concerns, there are no unique safety concerns for Pokémon Go players. Draw a line connecting (14A) and (15D). Without any support in the answers from Q15, (14B), (14C), and (14D) can be eliminated. The correct answers are (14A) and (15D).

15.D

(See explanation above.)

16.C

The question asks which park had fewer visits than Olympic National Park in 2018, according to the graph. First locate the number of visits to Olympic National Park in 2018. The right side of the graph shows that Olympic National Park had 3,104,455 visits in 2018. There is only one park that had fewer visits in 2018: Glacier National Park. The correct answer is (C).

17.D

The question asks which statement best matches the structure of the passage. Since this is a general question, it should be answered after the specific questions. The passage begins by stating, Coral reefs are among the most valuable natural assets on Earth and then discusses the role that herbivorous fish play in maintaining coral reefs. Eliminate answers that don’t match the prediction. Though the authors do discuss various types of herbivorous fish, they do not discuss how these types interact with each other. Eliminate (A). Though the text mentions herbivorous fish several times, the passage is not about the history of research on herbivorous fish. Eliminate (B). No specific event that occurs on coral reefs is discussed in depth in the passage. Eliminate (C). Keep (D) because it refers to both herbivorous fish and coral reefs and matches the prediction. The correct answer is (D).

18.C

The question asks for the authors’ perspective on studying coral reefs. Since there is no line reference, use lead words and the order of the questions to find the window. There are no line references prior to this question, but Q19 asks about line 5, so the window for Q18 is likely in the first paragraph. Scan the first paragraph, looking for the lead words coral reefs. The passage begins by stating, Coral reefs are among the most valuable natural assets on Earth. The correct answer should reflect the authors’ belief in the importance of coral reefs. Eliminate answers that don’t match the prediction. The passage says that seaweeds and algal turfs grow on coral reefs and that herbivorous fish eat them, but it does not indicate that the authors find it confusing that there are varying amounts of seaweed and algal turfs. Eliminate (A). The authors mention that populations of herbivorous fish vary widely and explore the reasons behind the variations, but the passage does not say that the topic is difficult to understand. Eliminate (B). Choices (A) and (B) are Mostly Right, Slightly Wrong trap answers. Choice (C) is supported because the text describes coral reefs as valuable and mentions the importance of coral reef conservation. Keep (C). While the authors do discuss human impacts on coral reefs, no mention is made of any controversy. Eliminate (D). The correct answer is (C).

19.C

The question asks what the word shocks most nearly means in line 5. Go back to the text, find the word shocks, and cross it out. Carefully read the surrounding text to determine another word that would fit in the blank based on the context in the passage. The text states that reefs face many stresses and shocks and then provides several examples of those shocks: overfishing, habitat damage, and pollution. Therefore, shocks must mean something like “problems” or “disruptions.” Surprises does not match “problems,” so eliminate (A). Jolts does not match “problems.” Eliminate (B). Disturbances matches “problems,” so keep (C). Reactions does not match “problems,” so eliminate (D). The correct answer is (C).

20.B

This question asks why conservation efforts should take local conditions into account, based on evidence about populations of herbivorous fish. This is the first question in a paired set, but it is easy to find, so it can be done on its own. Since there is no line reference, use lead words and the order of the questions to find the window. Q18 and Q19 ask about the first paragraph, so the window for Q20 most likely comes after the first paragraph. Beginning with the second paragraph, scan the passage looking for the lead words local conditions, which appear in line 29. Earlier in that paragraph, the authors state, we found that populations of herbivorous fish vary widely from site to site, and are strongly influenced by factors including temperature and island type. Thus, local conditions are important because they can determine the populations of herbivorous fish. Eliminate answers that don’t match the prediction. Choice (A) is a Could Be True trap answer, but no competing species are mentioned in this paragraph, so eliminate (A). Choice (B) is well supported by the prediction, so keep it. Choice (C) is a Right Words, Wrong Meaning trap answer. Though the authors mention herbivorous fish in this window, the text directly contradicts (C), as it indicates that herbivorous fish are strongly influenced by factors including temperature. Eliminate (C). No other important species are mentioned in this paragraph, so eliminate (D). The correct answer is (B).

21.B

The question is the best evidence question in a paired set. Because Q20 was easy to find, simply look at the lines used to answer the previous question. The prediction for Q20 was found in lines 24—27, which state, we found that populations of herbivorous fish vary widely from site to site, and are strongly influenced by factors including temperature and island type. These lines match those in (B). The correct answer is (B).

22.A

The question asks what the word drivers suggests about the fish population. Use given line reference to find the window. In the window, the authors state that it is hard to separate human impacts from the natural biophysical and environmental drivers of these fish populations. Therefore, the authors imply that the fish population is impacted by both human and natural factors. Eliminate answers that don’t match the prediction. Choice (A) says that human and natural factors affect the fish population. This matches the prediction, so keep (A). Choice (B) is a Mostly Right, Slightly Wrong trap answer, as the feeding habits of fish are not mentioned in the window. Eliminate (B). Choice (C) is a Right Words, Wrong Meaning trap answer. Though the words fishing and fish populations appear in the passage, there are no new locations around the globe mentioned in the window. Eliminate (C). Choice (D) is a Right Words, Wrong Meaning trap answer. The text mentions human impacts and states that those impacts are hard to separate from other factors that affect fish populations. This does not mean that human activity has not caused a significant change in fish populations, but that it is hard to determine the source of the change. Eliminate (D). The correct answer is (A).

23.C

The question is a best evidence question that asks for evidence that supports the claim that different types of herbivorous fish perform different roles in the reef environment. Use the line references given in the answer choices to find a statement that supports this claim. The correct answer must discuss both different types of fish and different roles. Choice (A) does not mention types of fish, so eliminate it. Choice (B) states that all herbivorous fish are not equal, but this information does not specify in what way they aren’t equal and does not relate to the fish’s roles. Eliminate (B). Choice (C) mentions specific types of herbivorous fish (browsers, detritivores, and scrapers) and their roles, so keep (C). Choice (D) mentions only one type of fish, so eliminate it. The correct answer is (C).

24.D

The question asks which group includes the greatest biomass of herbivorous fish. Work through each answer choice using the figures. Figure 2 shows a biomass of about 1.8 for detritivores at a water temperature of 28 degrees Celsius. Figure 1 shows a biomass close to 1.5 for parrotfishes when there are 0 humans per hectare of reef. Figure 1 shows a biomass close to 0 for browsers and parrotfishes when there are 40 humans per hectare of reef. Figure 2 shows a biomass of about 3 for browsers when the water temperature is 20 degrees Celsius. Since browsers when the water temperature is 20 degrees Celsius have the highest biomass among the answer choices, keep (D) and eliminate (A), (B), and (C). The correct answer is (D).

25.B

The question asks which idea from the passage is supported by the information in the graphs. Work through each answer choice using the figures. Eliminate (A), as different types of seaweed are not mentioned in either figure. Choice (B) states, Certain types of herbivorous fish are affected by the presence of humans. Human presence is measured on the horizontal axis of figure 1, and the graph shows that as the number of humans increases, the number of browsers and parrotfishes generally declines. The idea in (B) also appears in the passage, which mentions fishing as causing widespread reductions in herbivorous fish populations. Keep (B). Eliminate (C) because neither figure measures different functional roles in coral reefs. The figures do not measure the relationship between conservation and fish populations, so eliminate (D). The correct answer is (B).

26.A

The question asks for the main purpose of the passage. Since this is a general question, it should be answered after the specific questions. The blurb states that Nehru delivered this speech to India’s Parliament. Throughout the speech, Nehru talks about the responsibility that rests upon the Parliament and the need to work hard to give reality to our dreams for India. Therefore, the speech is intended to encourage the members of Parliament to work hard, to fulfill their responsibilities, and to help India succeed. Eliminate answers that don’t match the prediction. Keep (A) because it matches the prediction. Eliminate (B) because the speech discusses the need for Parliament to create new opportunities, rather than for Indian citizens to take advantage of new opportunities. Choice (C) is a Right Answer, Wrong Question trap answer—it includes a detail from the passage, but it doesn’t answer the question that was asked. In the second paragraph, Nehru does say that our hearts are heavy with the memory of the sorrow that people suffered before gaining independence. However, this is only mentioned once in the passage, so comforting those who lost a great deal is not the main purpose of the passage. Eliminate (C). Eliminate (D) because Nehru does not seek advice from the Parliament; instead he tells them what he thinks is most important. The correct answer is (A).

27.D

The question asks for a summary of the passage. Since this is a general question, it should be answered after the specific questions. The blurb indicates that Nehru delivered this speech to India’s Parliament when India was about to become independent from the United Kingdom. In the first paragraph, Nehru states, The achievement we celebrate today is but a step, an opening of opportunity, to the greater triumphs and achievements that await us. Are we brave enough and wise enough to grasp this opportunity and accept the challenge of the future? Throughout the speech, Nehru talks about the hard work that needs to be done to give reality to our dreams. Therefore, the speech celebrates independence but also conveys that there is more work to be done. Eliminate answers that don’t match the prediction. Eliminate (A) because Nehru does not focus on encouraging the common people to organize; instead, he focuses on the Parliament’s responsibility to create opportunity for India’s people. Eliminate (B) because the passage focuses on India at a particular moment in history, not on the nations of the world throughout history. Although Nehru mentions that nations must work together, this is one detail in the speech, and therefore it does not summarize the passage, making it a Right Answer, Wrong Question trap answer. Eliminate (C) because the tone of the passage is hopeful. Although sorrow is mentioned in one paragraph, Nehru does not indicate that the Indian people will be sorrowful and burdened until they realize their dreams. Keep (D) because it matches the prediction. The correct answer is (D).

28.A

The question asks what Nehru wants to convey with the statement in lines 4—6. Use the line reference to find the window. This sentence states that India has never lost sight of its goals or forgotten the ideals which gave it strength, through both good and ill fortune. Nehru uses this statement to convey that India has been true to its ideals and goals in both good and bad circumstances. Eliminate answers that don’t match the prediction. Choice (A) matches the prediction, so keep it. Eliminate (B) because the lines in the question do not make any prediction for the future; later in the paragraph, Nehru indicates that India will succeed, but that idea is not conveyed by these lines. Therefore, (B) a Right Answer, Wrong Question trap answer that is based on a fact from the passage but does not answer the question that was asked. Eliminate (C) because the lines in the question do not mention any of India’s leaders. Choice (C) is a Could Be True trap answer that is not directly supported by the text. Eliminate (D) because in these lines, Nehru does not suggest what India’s people must do; instead, he describes what they have already done. This is a Mostly Right, Slightly Wrong trap answer. The correct answer is (A).

29.D

The question asks why Nehru uses the words birth and pains. Use the given line reference to find the window. In lines 16—17, Nehru uses a metaphor of giving birth to refer to the difficulties that the Indian people faced before they achieved independence. Like giving birth, the struggle for independence was painful, but the struggle produced something worthwhile. Eliminate answers that don’t match the prediction. Choice (A) says that sad memories will continue even in happier times, but Nehru does not say that the memories will continue in the future; he says that some of those sorrowful memories continue even now. Choice (A) is a Right Words, Wrong Meaning trap answer, so eliminate it. Although Nehru indicates that freedom is a positive outcome, he does not mention joy in the lines that the question asks about. Furthermore, (B) does not mention the struggles of the past, so it does not match the prediction. Eliminate (B). Eliminate (C) because Nehru does not indicate that there will be an easy road ahead. Keep (D) because it matches the prediction. The correct answer is (D).

30.C

The question asks what Nehru suggests that the members of Parliament must do to help India succeed. Notice that this is the first question in a paired set, so it can be done in tandem with Q31. Look at the answers for Q31 first. The lines for (31A) do not mention anything that members of Parliament must do. These lines do not address Q30, so eliminate (31A). The lines for (31B) say, That future is not one of ease or resting but of incessant striving so that we may fulfill the pledges we have so often taken and the one we shall take today. Nehru’s speech is directed to the members of India’s Parliament, and in this sentence, he tells them that they must work hard to fulfill the promises they have made. Check the answers for Q30 to see whether any of the answers are supported by these lines. This information supports (30C), so draw a line connecting (31B) with (30C). The lines for (31C) do not mention anything that members of Parliament must do. These lines do not address Q30, so eliminate (31C). The lines for (31D) say that India promises to cooperate with the peoples of the world to support peace, freedom and democracy. This information does not support any of the answers to Q30: although (30D) mentions keeping promises, they are promises that the common people have made to each other, which does not match the lines for (31D). Eliminate (30D). Without any support in the answers from Q31, (30A), (30B), and (30D) can be eliminated. The correct answers are (30C) and (31B).

31.B

(See explanation above.)

32.C

The question asks for Nehru’s perspective on opportunity. Since there is no line reference, use lead words and the order of the questions to find the window. The answer to Q31 came from lines 21—23, so the window for Q32 is most likely after those lines. Scan the passage beginning with line 24, looking for the lead word opportunity. Lines 24—26 state, The service of India…means the ending of…inequality of opportunity. Nehru is speaking to the members of the Indian Parliament, so this statement indicates that he believes the government should support equal opportunity for India’s people. Eliminate answers that don’t match the prediction. Eliminate (A) because it contradicts the prediction, saying that there is no need to work for greater opportunity. Eliminate (B) because Nehru does not discuss balancing opportunities for personal prosperity with opportunities for national prosperity. Keep (C) because it matches the prediction. Eliminate (D) because Nehru does not contrast the importance of economic opportunity and religious equality. The correct answer is (C).

33.C

The question asks what the word narrow most nearly means in line 68. Go back to the text, find the word narrow, and cross it out. Then read the window carefully, using context clues to determine another word that would fit in the text. The text says, We cannot encourage…narrow-mindedness, for no nation can be great whose people are narrow in thought or in action. Therefore, narrow must mean something like “restricted,” and must refer to thoughts and actions. Thin does not match “restricted,” so eliminate (A). Fine does not match “restricted,” so eliminate (B). Limited matches “restricted,” so keep (C). Although a space that is tight might also be described as “restricted,” this refers to physical space, not to thoughts and actions, so eliminate (D). Note that (A), (B), and (D) are Could Be True trap answers based on other meanings of narrow that are not supported by the text. The correct answer is (C).

34.B

The question asks what Nehru suggests is most important to support India in reaching its potential. Since there is no line reference, use the order of the questions to find the window. Q33 asked about line 68, so the window for Q34 is most likely after that line. Scan the final paragraph for information about what Nehru thinks is important to support India. Lines 72—75 say, And to India…we pay our reverent homage and we bind ourselves afresh to her service. Nehru thinks that is important to respect and serve India (reverent means “highly respectful” and homage means “show of respect”). Eliminate answers that don’t match the prediction. Nehru does not say that evaluating good and ill fortune is important; the words good and ill fortune appear in the first paragraph, but there is no mention of evaluating them to support India. Eliminate (A). Keep (B) because it matches the prediction. Choice (C) is a Right Words, Wrong Meaning trap answer: in the first paragraph, Nehru mentions ideals, but says that they have never been forgotten. Therefore, remembering forgotten ideals does not match the text. Eliminate (C). In the final paragraph, Nehru mentions other nations, but he does not say that India should seek their advice. Therefore, (D) is another Right Words, Wrong Meaning trap answer; eliminate (D). The correct answer is (B).

35.A

The question asks what can be reasonably inferred about neutrinos from Passage 1. Notice that this is the first question in a paired set, so it can be done in tandem with Q36. Look at the answer choices for Q36 first. The lines for (36A) say that neutrinos were predicted to be completely massless, but surprisingly do have some mass. Check the answers for Q35 to see whether any of the answers are supported by these lines. These lines do not support any of the answers for Q35, so eliminate (36A). The lines for (36B) indicate that the maximum weight of a neutrino was estimated to be at least six million times lighter than an electron. It is not possible to feel the weight of an electron, let alone an item lighter than an electron, so (35A) may be inferred from (36B). Draw a line connecting (36B) and (35A). The lines for (36C) say that scientists can infer the mass of a neutrino by measuring the energy of the electron. Both (36C) and (35D) discuss the size of a particle and its relation to other particles. However, the lines for (36C) discuss how the mass of a neutrino depends on the energy of the electron, whereas (35D) suggests that the charge of other particles depends on the size of the neutrino. Eliminate (36C). The lines for (36D) say that all ways of measuring mass will be used in finding a finite answer about the neutrino’s mass. These lines do not support any of the answers for Q35, so eliminate (36D). Without any support in the answers from Q36, (35B), (35C), and (35D) can be eliminated. The correct answers are (35A) and (36B).

36.B

(See explanation above.)

37.B

The question asks why the author uses the word weirdest in line 1. Use the given line reference to find the window. The first paragraph of Passage 1 says that neutrinos are nearly massless, and it discusses how a prior belief about the weight of neutrinos was surprisingly overturned. The paragraph suggests that not much is known about neutrinos, as the true weight of neutrinos has remained a mystery. Therefore, the correct answer should indicate that neutrinos are perplexing to scientists. Eliminate answers that don’t match the prediction. Though the paragraph discusses previous observations, the word weirdest is not used to describe these observations. Eliminate (A) and (D). The word weirdest is used to point out the puzzling nature of neutrinos. Keep (B). Choice (C) is a Mostly Right, Slightly Wrong trap answer: the word unusual could match the word weirdest, but (C) uses unusual to refer to recent experiments, not neutrinos. Eliminate (C). The correct answer is (B).

38.C

The question asks what the word robust most nearly means in line 42. Go back to the text, find the word robust, and cross it out. Then read the window carefully, using context clues to determine another word that would fit in the text. The text says that combing all the possible ways of measuring the neutrino mass will allow scientists to have a finite and robust answer. Therefore, robust must mean something like “dependable” or “consistent.” Exact means “precise,” which does not match “dependable,” so eliminate (A). Healthy does not match “dependable,” so eliminate (B). Reliable matches “dependable,” so keep (C). Vigorous means “energetic” or “strong” and does not match “dependable,” so eliminate (D). Note that (B) and (D) are Could Be True trap answers based on other meanings of robust that are not supported by the text. The correct answer is (C).

39.D

The question asks about the purpose of the last paragraph of Passage 2. Read the last paragraph of Passage 2 as the window. The paragraph discusses planned improvements to KATRIN and the measurements that may be possible as a result of the improvements: During the next five years, Drexlin’s collaboration plans to make continuous improvements to KATRIN’s sensitivity that could enable it to make an actual measurement of a neutrino’s mass—or to narrow the range of the estimate as far as the machine’s sensitivity will allow (lines 81—86). Eliminate answers that don’t match the prediction. The paragraph does not discuss the claim that scientists have studied particles for many years. Eliminate (A). The paragraph does not explain the methods used to measure neutrinos, so eliminate (B). The paragraph does not celebrate achievements; it discusses potential future achievements, so eliminate (C). The paragraph does discuss future research possibilities, so keep (D). The correct answer is (D).

40.A

The question asks for the main purpose of both passages. Because this is a question about both passages, it should be done after all the questions that ask about each passage individually. Passage 1 discusses a discovery that challenged a previous assumption about neutrinos and details future projects that will try to confirm this discovery. Passage 2 discusses one such project, the KATRIN experiment, and a discovery about neutrinos made from this experiment. Choice (A) matches the main focus of both passages, which is to discuss discoveries about one type of particle, the neutrino. Keep (A). Neither passage discusses how researchers developed the tools needed to make the calculations, so eliminate (B). Choice (C) is a Mostly Right, Slightly Wrong trap answer; a neutrino is a subatomic particle, but it is only one type of subatomic particle. Neither passage summarizes what physicists have learned about subatomic particles in general. Eliminate (C). Both passages indicate that scientists want to make more accurate measurements of the neutrino’s mass, but neither author makes an argument intended to persuade scientists. Eliminate (D). The correct answer is (A).

41.D

The question asks for a statement that the authors of Passage 1 and Passage 2 would agree on. Because this is a question about both passages, it should be done after the questions that ask about each passage individually. Notice that this is the first question in a paired set, so it can be done in tandem with Q42. Look at the answer choices for Q42 first. The lines for (42A) read, An experiment in Germany has made the most precise measurement yet of the maximum mass of neutrinos. Check the answers for Q41 to see if any of the answers are supported by these lines. These lines do not support any of the answers for Q41, so eliminate (42A). The lines for (42B) indicate that the new measurement is an improvement over previous measurements. These lines do not support any of the answers for Q41, so eliminate (42B). The lines for (42C) discuss the evolving states that a subatomic particle may assume as it decays before a neutrino is produced. This matches the idea that subatomic particles do not always keep the same form. Draw a line connecting (42C) and (41D). The lines for (42D) state that KATRIN cannot detect the neutrinos directly. Choice (41B) is a Right Words, Wrong Meaning trap answer related to these lines: KATRIN uses indirect measurements, but this does not indicate that indirect measurements will replace direct observation. Eliminate (42D) because these lines do not support any of the answers for Q41. Without any support in the answers from Q42, (41A), (41B), and (41C) can be eliminated. Because this pair is the only match, there is no need to find evidence in Passage 1. However, (41D) is supported in line 32 of Passage 1, which states that a neutron transforms into a proton. The correct answers are (41D) and (42C).

42.C

(See explanation above.)

Section 2—Writing and Language

1.B

Punctuation is changing in the answer choices, so this question is testing STOP, HALF-STOP, and GO punctuation. However, notice that punctuation is changing around three items in a list, so the question is also testing comma rules. In a list of three or more items, there needs to be a comma after each item in the list. Choice (A) has an extra comma after the word and, which is not correct, so eliminate (A). Choice (B) correctly uses a comma after every item in the list. Keep (B). Choices (C) and (D) each use a semicolon instead of a comma to separate two items in the list, which is not correct. Eliminate (C) and (D). The correct answer is (B).

2.C

The number of words is changing in the answer choices, so this question could be testing concision. Choices (A), (B), and (D) each include the phrase diamonds, rubies, sapphires, and compass, so determine whether this phrase is necessary. All of these items are mentioned in the previous sentence, and the word also implies that that the knowledge was carried along with those items, so there is no need to repeat the names of the items here. Eliminate (A), (B), and (D). The correct answer is (C).

3.C

Punctuation is changing in the answer choices, so this question is testing STOP, HALF-STOP, and GO punctuation. Use the Vertical Line Test and identify the ideas as complete or incomplete. Draw the vertical line between the words gained and from. The first part of the sentence, Polo also carried knowledge he had gained, is a complete idea. The second part, from his journey east, is an incomplete idea. To connect a complete idea to an incomplete idea, HALF-STOP or GO punctuation is needed. Eliminate (A) and (B) because a semicolon and a period are both STOP punctuation. Keep (C) because no punctuation is GO punctuation. The addition of the word it in (D) changes the second part of the sentence to a complete idea, it was from his journey east. Each part of the sentence is now a complete idea, so STOP punctuation is needed. Since a comma is GO punctuation, eliminate (D). The correct answer is (C).

4.C

Note the question! The question asks whether a sentence should be added, so it’s testing consistency. If the content of the new sentence is consistent with the ideas surrounding it, then it should be added. The paragraph discusses Marco Polo’s return from China and what he brought back. The new sentence discusses a belief held by Europeans, so it is not consistent with the ideas in the text; the sentence should not be added. Eliminate (A) and (B). Keep (C) because it accurately states that the new sentence interrupts the discussion of Marco Polo’s journey. Eliminate (D) because the information was not previously stated. The correct answer is (C).

5.D

The vocabulary and number of words are changing in the answer choices, so this question is testing precision of word choice and could be testing concision. The previous sentence states that Polo became appointed as a diplomat; it is redundant to restate as a diplomat because the word this clearly refers back to what he did as a diplomat, so eliminate (A) and (C). Between life and role, role is more precise and correctly refers to his appointment as a diplomat in the previous sentence. Eliminate (B). The correct answer is (D).

6.B

Note the question! The question asks whether a sentence should be revised, so it’s testing consistency. If the content of the revised sentence is consistent with the ideas surrounding it, then it should be revised. The original sentence says He encountered many things that were unknown in Europe. The proposed revision includes the specific things that Marco Polo encountered, so the sentence should be revised to make it more precise; eliminate (C) and (D). The revision doesn’t mention Polo’s everyday life in Asia, so eliminate (A). The revision does give details about what Marco Polo learned in his travels. The correct answer is (B).

7.A

Verbs are changing in the answer choices, so this question is testing consistency of verbs. A verb must be consistent with its subject and with the other verbs in the sentence. The subject of the verb is explanation, which is singular. To be consistent, the underlined verb must also be singular. Eliminate (B) and (D) because were and are are plural. The other verb in the sentence is was, which is in the past tense. To be consistent, the underlined verb must also be in the past tense. Eliminate (C) because it is not in the past tense. The correct answer is (A).

8.D

Note the question! The question asks where sentence 3 should be placed, so it’s testing consistency of ideas. The sentence must be consistent with the ideas that come both before and after it. Sentence 3 says that Polo’s explanation of paper money…was new and intriguing, so it must come after some mention of paper money. Sentence 5 discusses paper currency. Therefore, sentence 3 should follow sentence 5. The correct answer is (D).

9.D

Note the question! The question asks which choice would most effectively introduce the topic of the paragraph, so it’s testing consistency of ideas. Determine the subject of the paragraph and find the answer that is consistent with that idea. The paragraph discusses Polo’s original manuscript and says it may never be known exactly what Polo’s original manuscript said. Eliminate (A) and (B) because Christopher Columbus and the Age of Discovery are not mentioned in the paragraph. Eliminate (C) because the paragraph doesn’t mention historians questioning the accuracy of Polo’s manuscript. Keep (D) because it is consistent with the content of the paragraph. The correct answer is (D).

10.A

Punctuation is changing in the answer choices, so this question is testing STOP, HALF-STOP, and GO punctuation. Use the Vertical Line Test and identify the ideas as complete or incomplete. Draw the vertical line between the words including and cartography. The first part of the sentence, While it may never be known exactly what Polo’s original manuscript said, his influence lives on in many areas, including, is an incomplete idea. The second part, cartography, currency, and exploration, is an incomplete idea. To connect an incomplete idea to an incomplete idea, GO punctuation is needed. Keep (A) because no punctuation is GO punctuation. Eliminate (B) and (C) because a single dash and a colon are HALF-STOP punctuation. Eliminate (D) because a semicolon is STOP punctuation. The correct answer is (A).

11.D

Punctuation is changing in the answer choices, so this question is testing STOP, HALF-STOP, and GO punctuation. Use the Vertical Line Test and identify the ideas as complete or incomplete. Draw the vertical line through the FANBOYS word and between the words learning and the. The first part of the sentence, Research has shown that both terrestrial and aquatic animals practice spatial learning, is a complete idea. The second part, the ability to navigate a physical environment, is an incomplete idea. To connect a complete idea to an incomplete idea, HALF-STOP or GO punctuation is needed. Eliminate (A) and (B) because a comma with FANBOYS and a period are both STOP punctuation. Although no punctuation is GO punctuation, the two ideas need to be separated because the second part is an explanation of the first part; eliminate (C). Keep (D) because a single dash is HALF-STOP punctuation. The correct answer is (D).

12.C

Note the question! The question asks which choice provides the best transition to the next paragraph, so it’s testing consistency of ideas. Determine the subject of the paragraph and find the answer that is consistent with that idea. The paragraph discusses a study on how members of an aquatic species navigate an underwater environment. Eliminate (A) because the study is only on an aquatic species and does not include any terrestrial species. Eliminate (B) because the study is on only one aquatic species, not multiple. Keep (C) because it is consistent with the paragraph. Eliminate (D) because the study is on how the crabs navigate, and not why they need those abilities. The correct answer is (C).

13.C

Pronouns and apostrophes are changing in the answer choices, so this question is testing consistency of pronouns and apostrophes. In this sentence, the underwater environment belongs to members of an aquatic species, so a possessive pronoun is needed. A pronoun with an apostrophe is a contraction. They’re means “they are.” It is not correct to say that “they are” underwater environment; eliminate (A) and (B). Keep (C) because it uses the correct possessive pronoun to show that the underwater environment belongs to the crabs. Eliminate (D) because there is not a possessive pronoun. The correct answer is (C).

14.A

Note the question! The question asks whether a quotation should be removed, so it’s testing consistency. If the content of the quotation is consistent with the ideas surrounding it, then it should be kept. The paragraph discusses a study about the spatial abilities of shore crabs. The quotation gives a reason for the study, saying we haven’t really looked for them in their aquatic counterparts. Since the quotation is consistent with the rest of the paragraph, it should be kept; eliminate (C) and (D). Keep (A) because it correctly says that the quotation gives a reason for the study. Eliminate (B) because the quotation does not discuss the study’s scientific approach. The correct answer is (A).

15.A

The word after crabs is changing in the answer choices, so this question is testing precision and concision. First determine whether any of the words after crabs is necessary. The non-underlined portion of the sentence includes the word suspect and the phrase may have, so there is no need to further emphasize that Pope’s statement (crabs may have…abilities) is a theory. Eliminate (B), (C), and (D) because perchance, perhaps, and possibly are all redundant. The correct answer is (A).

16.B

Vocabulary is changing in the answer choices, so this question is testing precision of word choice. Look for a phrase with a definition that is consistent with the other ideas in the paragraph. The paragraph is about the study of the crabs. The previous sentence mentions the time frame four-week period. Since the first part of the sentence with the underlined portion describes an event that happened after the event in the previous sentence, the correct phrase must also refer to a time period. Eliminate (A) and (D) because weaks means “feeble” and is not a time period. Keep (B) because two weeks describes a time period. Eliminate (C) because too means “also” or “excessive.” The correct answer is (B).

17.B

Pronouns are changing in the answer choices, so this question is testing consistency of pronouns. A pronoun must be consistent in number with the noun it refers to. The underlined pronoun refers to the noun crabs, which is plural. To be consistent, the underlined pronoun must also be plural. Eliminate (A) because you is not consistent with crabs. Keep (B) because they is plural. Eliminate (C) and (D) because he or she and it are both singular. The correct answer is (B).

18.C

Note the question! The question asks where sentence 1 should be placed, so it’s testing consistency of ideas. The sentence must be consistent with the ideas that come both before and after it. Sentence 1 says that At the end of this period, scientists had observed a decrease in both the time it took each of the 12 crabs to complete the maze and the number of wrong turns each crab took, so it must come after some mention of a period of time. Sentence 3 mentions a four-week period. Therefore, sentence 1 should follow sentence 3. The correct answer is (C).

19.D

Vocabulary is changing in the answer choices, so this question is testing precision of word choice. Look for a phrase with a definition and tone that are consistent with the other ideas in the sentence. The sentence suggests that Pope “wants to do” further research. Choice (A) is too strong for the tone of the passage, as nothing suggests that Pope is desperate, so eliminate (A). Choice (B) uses the awkward phrase dreams with hope, which does not match the straightforward, factual tone of the passage, so eliminate (B). Choice (C) is also not consistent with the passage’s factual tone, so eliminate (C). Choice (D) is precise and consistent with the passage’s tone. The correct answer is (D).

20.B

Words are changing in the answer choices, so this question is testing consistency and precision of word choice. Look for a phrase that is consistent with the other ideas in the sentence. The previous sentence does not set up a contrast, so eliminate (A). Keep (B) because it clearly sets up the rest of the sentence, which summarizes the main idea of the passage. Both (C) and (D) imply that the crabs’ abilities are in some way caused by the research, which is not correct, so eliminate (C) and (D). The correct answer is (B).

21.D

Vocabulary is changing in the answer choices, so this question is testing precision of word choice. Look for a word with a definition that is consistent with the other ideas in the sentence. The sentence says that robotic surgery allows the doctor to perform procedures without making significant incisions. Procedures that would otherwise require significant incisions are complicated, so the correct word should mean “complicated.” Puzzling is similar in meaning to “complicated,” so keep (A). Twisted means “distorted,” which doesn’t match “complicated,” so eliminate (B). Mysterious means “secretive,” which doesn’t match “complicated,” so eliminate (C). Complex means “complicated,” so keep (D). Although puzzling has some support, it implies that the procedures are confusing to the doctors, which is not necessarily true. Eliminate (A). Choice (D) makes the meaning of the sentence most precise. The correct answer is (D).

22.A

Apostrophes are changing in the answer choices, so the question is testing apostrophe usage. When used with a noun, on the PSAT, an apostrophe indicates possession. In this sentence, the bodies belong to patients, so an apostrophe is needed after patients, and because patients is plural, the apostrophe should be placed after the s. Keep (A) because it supplies the apostrophe after the s in patients. Eliminate (B), (C), and (D) because none of these uses the correct apostrophe after patients. The correct answer is (A).

23.C

Note the question! The question asks how to effectively combine the underlined sentences, so it’s testing precision and concision. The phrase after industry changes in the answer choices, so determine the function of this phrase. The phrase must connect two sentences, so determine the relationship between the sentences. Sentence 1 says that surgical robots have changed the medical industry. Sentence 2 explains how they have changed the medical industry. Therefore, the underlined portion must give a clue that the information that follows explains a point from sentence 1. Eliminate (A) because the word though indicates that the information that is to follow will contrast sentence 1. Eliminate (B) because which suggests that surgical robots changing the medical industry caused doctors to be able to make precise cuts, which reverses the correct order of ideas. Keep (C) because by indicates that the information that follows will explain how surgical robots changed the medical industry. Eliminate (D) because that simply implies that the medical industry allows doctors to make precise cuts—this may be true, but it does not explain how surgical robots changed the industry. The correct answer is (C).

24.C

Verbs are changing in the answer choices, so this question is testing consistency of verbs. A verb must be consistent with its subject and with the other verbs in the sentence. The subject of the verb is physician, which is singular. To be consistent, the underlined verb must also be singular. Eliminate (A) and (D) because have been and were are plural. Verbs must also be consistent with other verbs in the sentence. The other verb in the sentence, before the underlined portion, is makes, which is in the present tense. To be consistent, the underlined verb must also be in the present tense. Eliminate (B) because was is past tense. Keep (C) because it is present tense. The correct answer is (C).

25.B

Note the question! The question asks which choice most accurately represents the data in the chart, so it’s testing consistency. Read the labels on the chart carefully, and look for an answer that is consistent with the information given in the chart. Notice that all four answer choices involve the number 86, so look for this number on the chart and determine its meaning. The chart shows 86% for faculty-led time with robot, and the title indicates that the percentages are the portion of medical programs that train doctors using these techniques. Eliminate (A) because it does not relate to faculty-led time. The chart says that 86 percent of medical programs use faculty-led time with a robot to train students, so (B) is consistent with the chart. Keep (B). The chart does not show how much time students spent with robots, so (C) is not consistent with the chart. Eliminate (C). Choice (D) implies case observation rather than faculty-led time with robot, which was used by 80% of programs, not 86%. Eliminate (D). The correct answer is (B).

26.A

Note the question! The question asks which choice most accurately and precisely provides specific data from the chart, so it’s testing consistency. Read the labels on the chart carefully, and look for an answer that is consistent with the information given in the chart. The chart says that 51 percent of programs used videos to train students, so (A) is consistent with the chart. Keep (A). Since the bars represent percentages, not the number of programs, it is not true that 51 programs used videos to train students. Eliminate (B). It is true that some of the participants in the survey (the medical programs) used videos to train students, so keep (C). Likewise, it is also true that additional programs used videos, to add on to the methods mentioned in the previous sentence. While (A), (C), and (D) are all true, the question is asking for specific data. Choice (A) provides a specific percentage, so eliminate (C) and (D). The correct answer is (A).

27.A

Commas are changing in the answer choices, so this question is testing the four ways to use a comma. Commas are changing around the phrase to an effective local surgeon, so determine whether this phrase is necessary. The phrase identifies the people that an isolated or impoverished region might not have access to, so it is necessary to the meaning of the sentence. The phrase should not be surrounded by commas. However, the sentence is constructed with an incomplete idea followed by a complete idea, and a comma is needed between the two ideas to separate them. So, there should not be a comma before the phrase, but there should be a comma after it. Keep (A) because it does not place a comma before the phrase, but it does place a comma after the phrase. Eliminate (B) because it places a comma before and after the phrase. Eliminate (C) because it does not place a comma after the phrase. Eliminate (D) because it places a comma before the phrase. The answer correct is (A).

28.D

Transitions are changing in the answer choices, so this question is testing consistency of ideas. A transition must be consistent with the relationship between the ideas it connects. The sentence before the transition states that A 2017 study…showed a 95 percent overall success rate for robotic surgeries, and the sentence that starts with the transition states that the same study showed that no patients had complications from the surgeries mentioned in the previous sentence. These ideas agree, so a same-direction transition is needed. Surprisingly implies that the second sentence will provide information that is unexpected to follow the first sentence. Since both ideas are positive, this is not the correct direction. Eliminate (A). The two sentences agree, so eliminate (B), which contains an opposite-direction transition. As previously stated indicates that the positive recoveries of patients was previously mentioned. This is not true, so eliminate (C). The second sentence provides an additional outcome of the study mentioned in the first sentence, so additionally is an appropriate link between the two ideas. Keep (D). The correct answer is (D).

29.C

Transitions are changing in the answer choices, so this question is testing consistency of ideas. A transition must be consistent with the relationship between the ideas it connects. The sentence before the transition states that Surgical robots can help doctors to make precise movements in complex procedures, and the sentence that starts with the transition states that surgical robots also allow doctors to operate remotely. These two sentences each discuss a benefit of surgical robots, so the correct transition must indicate that the second sentence is an additional benefit. For this reason implies that the first sentence provides a reason that doctors may operate remotely. This is not true, so eliminate (A). The two sentences agree, so eliminate (B), which contains an opposite-direction transition. As another benefit indicates that the second sentence will contain an additional benefit of robotic surgery. This is true, so keep (C). Thus implies that the second sentence is a conclusion that follows from the first sentence. This is not true, so eliminate (D). The correct answer is (C).

30.D

Prepositions are changing in the answer choices, so this question is testing idioms. Look at the phrase before the preposition to determine the correct idiom. The first half of the sentence says that some patients need surgery right away, and the word Since implies that the information after the comma will provide a conclusion that relates to the first half. The second half of the sentence implies that surgical robotics can save the lives of those who need surgery right away but don’t have access to an on-site doctor. Therefore, surgical robotics connects people through the use of life-saving technology. Eliminate (A) because while makes the sentence incomplete. Choice (B) implies that people connect the field of surgical robotics, which is the opposite of what the sentence should say. Eliminate (B). Eliminate (C) because the phrase connecting with people with…technology is not correct. Choice (D) suggests that surgical robotics can connect people with life-saving technology. This is true, and it is the correct meaning of the sentence in context. The correct answer is (D).

31.C

Punctuation is changing in the answer choices, so this question is testing STOP, HALF-STOP, and GO punctuation. Use the Vertical Line Test and identify the ideas as complete or incomplete. Draw the vertical line between the words gathering and on. The first part of the sentence, The crowd was gathering, is a complete idea. The second part, on September 3, 1922, as Bessie Coleman looked up to see a clear sky, ideal conditions for her air show, is an incomplete idea. To connect a complete idea to an incomplete idea, HALF-STOP or GO punctuation is needed. All the answer choices are either HALF-STOP or GO punctuation, so look for another way to eliminate choices. Although a comma is GO punctuation, there is no reason to use a comma after gathering. Eliminate (A) and (D). A colon implies that the following information is either a list or an explanation of a prior idea. Since neither happens in this sentence, there is no reason to use a colon after gathering. Eliminate (B). The correct answer is (C).

32.C

Transitions are changing in the answer choices, so this question is testing consistency of ideas. A transition must be consistent with the relationship between the ideas it connects. The sentence before the transition states that Bessie had fallen in love with flying years before the air show while watching newsreels about World War I, and the sentence that starts with the transition discusses how aviation technology changed during a time of war. Since both sentences indicate actions that occurred continuously during the war, the transition must indicate a general period of time. At this point and At this time both indicate a specific time, not a general period, so eliminate (A) and (B). To choose between (C) and (D), consider the idiom being used. Time of war is the correct idiom in this context, so keep (C) and eliminate (D). The correct answer is (C).

33.D

Pronouns are changing in the answer choices, so this question is testing consistency of pronouns. A pronoun must be consistent in number with the noun it refers to. The underlined pronoun refers to the noun aspiring aviators, which is plural. To be consistent, the underlined pronoun must also be plural. Eliminate (B) and (C) because she and he are singular pronouns. The pronouns you and they can be used in plural contexts, so consider the other pronouns in the sentence. The sentence uses they elsewhere to refer to aspiring aviators, so the underlined pronoun must be consistent. Eliminate (A). The correct answer is (D).

34.B

Commas are changing in the answer choices, so this question is testing the four ways to use a comma. Commas change around the phrase who would train Bessie, so determine if the phrase is necessary. The phrase is necessary information, since it explains why the instructors were important, so it should not have commas around it. Choices (A), (C), and (D) all include commas, so eliminate them. The correct answer is (B).

35.D

Note the question! The question asks which choice would explain why Bessie might be cautious, so it’s testing consistency. Eliminate answers that are inconsistent with the purpose stated in the question. The paragraph discusses Bessie’s experience in flight school, and the sentence before question 35 states that Bessie knew that aviation training was dangerous. Look for an answer that explains how Bessie might have known it was dangerous. Eliminate (A) because the length of her training does not relate to the dangers of aviation training. Eliminate (B) because it discusses danger in the present day, not in Bessie’s time. Eliminate (C) because there is no indication that Bessie thought of danger while imagining herself in the pilot’s seat. Keep (D) because watching a fellow classmate die in a crash supports the idea that aviation training was dangerous. The correct answer is (D).

36.B

Note the question! The question asks where sentence 2 should be placed, so it’s testing consistency of ideas. The sentence must be consistent with the ideas that come both before and after it. Sentence 2 says that Bessie was, once again, unable to find an instructor in any American school, so it must come after some mention of a decision to return to school. Sentence 3 states that Bessie decided to pursue advanced aviation training. Therefore, sentence 2 should follow sentence 3. The correct answer is (B).

37.C

Note the question! The question asks whether a sentence should be added, so it’s testing consistency. If the content of the new sentence is consistent with the ideas surrounding it, then it should be added. The paragraph discusses Bessie’s career-defining air show at Curtiss Field. The new sentence discusses safety concerns about stunt flying after World War II and a contemporary effect of these consequences, so it is not consistent with the ideas in the text; the sentence should not be added. Eliminate (A) and (B). Keep (C) because it accurately states that the new sentence is irrelevant. Eliminate (D) because the paragraph does not focus on explaining airshows. The correct answer is (C).

38.D

The wording changes in the answer choices, so this question is testing consistency. There is also the option to DELETE; consider this choice carefully as it is often the correct answer. Determine whether the underlined phrase is necessary. The paragraph discusses Bessie’s career-defining air show at Curtiss Field. The first part of the sentence that contains the underlined phrase says that The crowd cheered loudly. Choices (A), (B), and (C) all give reasons that the crowd may have cheered for Bessie, but all of those reasons are implied by the statement that the audience cheered loudly. Eliminate (A), (B), and (C) because they are overly wordy. The correct answer is (D).

39.A

Vocabulary is changing in the answer choices, so this question is testing precision of word choice. Look for a word with a definition that is consistent with the other ideas in the sentence. The sentence says that Bessie toured the country, showcasing a range of impressive aerial maneuvers, so the correct word should mean “exciting.” Electrifying means “thrilling,” which matches “exciting.” Keep (A). Pleasing means “making happy,” which is similar to “exciting” but does not convey the same energy. Eliminate (B). Occupying means “keeping busy,” which does not match “exciting.” Eliminate (C). Growing means “making larger,” which does not match “exciting.” Eliminate (D). The correct answer is (A).

40.A

Note the question! The question asks which choice best emphasizes the idea of Bessie Coleman as a significant aviator, so it’s testing consistency of ideas. Keep (A) because the idea that “Queen Bess” continues to be an inspiration to people matches the idea that she was a significant aviator. Eliminate (B) because Coleman’s memories do not suggest her significance. Eliminate (C) because it focuses on the start of her career rather than the importance of her influence. Eliminate (D) because it focuses on one detail from the text rather than Coleman’s overall significance. The correct answer is (A).

Section 3—Math (No Calculator)

1.B

The question asks for a system of equations that represents a specific situation. Translate the question in bite-sized pieces and eliminate after each piece. One piece of information says the store sold 52 pieces of softball equipment, so one of the equations must equal 52. Eliminate (A) and (D), as neither choice shows an equation equaling 52. Compare the remaining answer choices. The difference between (B) and (C) is the right side of the second equation. Since the question states that gloves (g) cost $20 and bats (b) cost $50, one of the equations shows 50b and 20g being added together to equal the total money made by the store. Eliminate (C), as it does not show 50b + 20g as equal to the total of 1,700. The correct answer is (B).

2.C

The question asks for the value of a that satisfies an equation. There are specific values in the answers, so plug in the answers. Begin by labeling the answers as a, and start with (B), 24. Plug this value into the equation to see if it makes the equation true. The equation becomes or . This is not true, so eliminate (B). Since (B) is too small, (A) can also be eliminated. Plug (C), 48, into the equation. The equation becomes or . This is true, so stop here. Another way to approach this is by solving the equation for the value of a. To solve for a, cross-multiply to get (8) (12) = 2a or 96 = 2a. Divide both sides by 2 to get 48 = a. Either way, the correct answer is (C).

3.D

The question asks for an equation that represents the relationship between two variables. When given a table of values and asked for the correct equation, plug values from the table into the answer choices to see which one works. According to the table, f(x) = 2 when x = 0. Choice (A) becomes 2 = 4(0) — 2, or 2 = —2. This is not true, so eliminate (A). Choice (B) becomes 2 = —2(0) + 2, or 2 = 2. This is true, so keep (B) for now, but check the remaining answers just in case. Choice (C) becomes 2 = —3(0) + 1, or 2 = 1. This is not true, so eliminate (C). Choice (D) becomes 2 = —4(0) +2, or 2 = 2. This is true, so keep (D) for now. Since two answers match the target value, choose a new set of values from the table to plug into the remaining answer choices. According to the table, f(x) = —2 when x = 1. Choice (B) becomes —2 = —2(1) + 2, or —2 = 0. This is not true, so eliminate (B). Choice (D) becomes —2 = —4(1) + 2, or —2 = —2, which is true. The correct answer is (D).

4.A

The question asks for an equation that represents a line. To find the best equation, compare features of the line to the answer choices. The graph for this line has a y-intercept of —5 and a positive slope. Eliminate answer choices that do not match this information. The equations are written in slope-intercept form, y = mx + b, where m represents the slope and b represents the y-intercept. Choices (B) and (D) have negative slopes, so eliminate (B) and (D). Compare the remaining answer choices. Both (A) and (C) have y-intercepts of —5 and positive slopes, so find the slope of the line using two points on the line, such as (6, —1) and (3, —3). Slope is calculated using the equation . The equation becomes . Choice (C) has a slope of , so eliminate (C). Another option would be to pick a point on the graph such as (6, —1) and plug it into the given equations to see if they are true. Either way, the correct answer is (A).

5.B

The question asks for an equivalent form of an expression. There are variables in the answer choices, so plug in. Make t = 1 and v = 2. The expression becomes —9[3(1) — 3(2)] + [9(1) — 13(2)] = —9(3 — 6) + (9 — 26) = —9(—3) — 17 = 27 — 17 = 10. This is the target value; circle it. Now plug t = 1 and v = 2 into the answer choices to see which one matches the target value. Choice (A) becomes —18(1) —40(2) = —18 — 80 = —98. This does not match the target value, so eliminate (A). Choice (B) becomes —18(1) + 14(2) = —18 + 28 = 10. This matches the target value, so keep (B), but check the remaining answers, just in case. Choice (C) becomes 36(1) + 14(2) = 36 + 28 = 64. Eliminate (C). Choice (D) becomes 36(1) — 40(2) = 36 — 80 = —44. Eliminate (D). The correct answer is (B).

6.A

The question asks for an equivalent form of an expression. There is a variable in the answer choices, so plug in. Make c = 2. The expression becomes 3(2)2 — 10(2) + 8 = 3(4) — 20 + 8 = 12 — 20 + 8 = 0. This is the target value; circle it. Now plug c = 2 into the answer choices to see which one matches the target value. Choice (A) becomes [3(2) — 4][(2) — 2] = (2)(0) = 0. This matches the target value, so keep (A) for now, but check the remaining answers just in case. Choice (B) becomes [3(2) + 4] [(2) — 2] = (10)(0) = 0. Keep (B) for now. Choice (C) becomes [3(2) — 8][(2) + 1] = (—2)(3) = —6. This is does not match the target value, so eliminate (C). Choice (D) becomes [3(2) + 8][(2) + 1] = (14)(3) = 42. Eliminate (D). Since two answers match the target value, choose a new value to plug in to the remaining answer choices. Make c = 3. The expression becomes 3(3)2 — 10(3) + 8 = 3(9) — 30 + 8 = 27 — 30 + 8 = 5. This is the new target value; circle it. Choice (A) becomes [3(3) — 4][(3) — 2] = (5)(1) = 5. This matches the new target, so keep (A) for now. Choice (B) becomes [3(3) + 4][(3) — 2] = (13)(1) = 13. Eliminate (B). The correct answer is (A).

7.B

The question asks for the value of m that satisfies an equation. Since the question asks for a specific value and the answers contain numbers, Plugging In the Answers could work. However, the answers are in decimal form, so it may get confusing without a calculator. Try to simplify the equation instead. Start by distributing the negative sign and combining like terms. The equation becomes 8m — 3 + 2m — 6 = 10m — 9 = 0. Add 9 to both sides of the equation to get 10m = 9, then divide both sides by 10 to get m = = 0.9. The correct answer is (B).

8.D

The question asks for an equation in terms of a specific variable. Although there are variables in the answer choices, plugging in on this question would be difficult because of the number of different variables. Instead, solve for α. To begin to isolate α, subtract ω0 from both sides of the equation to get ωω0 = αt. Divide both sides by t to get . The correct answer is (D).

9.A

The question asks for the relationship between the heights of two cylinders of identical volume. The question involves a relationship between unknown numbers, so plug in. The question states the radius of Paperweight X is one-third the radius of Paperweight Y, so use rX = 3 for Paperweight X and rY = 9 for the Paperweight Y. The formula for the volume of a cylinder is V = πr2h, so plug in the values to determine the volume of each cylinder. The volume of the Paperweight X is V = π(3)2hX = 9πhX. The volume of Paperweight Y is V = π(9)2hY = 81πhY. Since the volumes are equal, set the equations equal to get 9πhX = 81πhY. Divide both sides by 9π to get hX = 9hY. Paperweight X’s height will be 9 times that of Paperweight Y. The correct answer is (A).

10.C

The question asks about the graph of the data representing a certain situation. Label the parts of the equation to determine what they represent. In this question, C represents the cost of living in the apartment for a month and h represents the number of kilowatt hours of electricity used in a month. The y-intercept of a graph is defined as a point where the graph intersects the y-axis, so the y-intercept must relate to cost of living in the apartment. Eliminate (A), since it does not mention the cost of rent. The equation is in y = mx + b form, where b is the y-intercept of a graph, so here the y-intercept is 950. The question states the only other apartment expense is $950 a month for rent. Therefore, the y-intercept represents the cost of rent. The correct answer is (C).

11.2

The question asks for the value of a variable in a system of equations. The point of intersection will be the x and y solutions to the system, so solve the system of equations using stacking and adding. To make the x terms disappear, multiply the second equation by —2. The second equation becomes —2(—4x — 2y) = —2 (3) or 8x + 4y = —6. Now stack the equations and add them together.

Divide both sides of the resulting equation by 7 to get y = 2. The correct answer is 2.

12. or 0.5

The question asks for the value of y-intercept of an equation. The y-intercept of a graph is defined as a point where the graph intersects the y-axis, or when x = 0. To find the y-coordinate at that point, plug x = 0 into the equation. The equation becomes y = —3(0)2 + (0) + , which simplifies to —3(0) + , or . The correct answer is or 0.5.

13.3

The question asks for the cost a special exhibit ticket when given information about the total cost of a trip to a museum. Translate the English to math in bite-sized pieces. The price of a general admission ticket is $10. There were 9 students on the trip, so the cost of general admission can be represented as 90. The price of a special exhibit ticket can be represented as s. The question states that 7 students bought 3 special exhibit tickets each, which can be represented as 21s. It also states that 2 students bought 4 special exhibit tickets each, which can be represented as 8s. The total cost was $177.00, so the information can be written as 177 = 90 + 21s + 8s or 177 = 90 + 29s. Subtract 90 from both sides of the equation to get 87 = 29s, then divide both sides by 29 to get s = = 3. The correct answer is 3.

Section 4—Math (Calculator)

1.C

The question asks for the number of paperclips that could be in 5 boxes. Since there is a range for the number of paperclips in one box, find the minimum and maximum number that could be in 5 boxes. If each box contains the minimum of 300 paperclips, 5 boxes would contain 5(300) = 1,500 paperclips. Eliminate (A) and (B), as these numbers are too small. If each box contains the maximum of 400 paperclips, 5 boxes would contain 5(400) = 2,000 paperclips. Eliminate (D), as this number is too large. The correct answer is (C).

2.A

The question asks for the number of full-time employees that were dissatisfied with their work schedules. Looking at the chart, the space under the column “Full-time” in the row “Dissatisfied” is the only box that is not filled in. Call this number x. Read the information above the chart to determine how many employees were surveyed. The question states that 500 employees were surveyed, so the numbers in the chart add up to 500, or 126 + 119 + 108 + x = 500. Simplify to 353 + x = 500, then subtract 353 from both sides to get x = 147. The correct answer is (A).

3.C

The question asks for the number of dry erase markers the teacher expects to use for a certain number of students. Begin by reading the question to find information about the number of red pens. The question states that the teacher uses 1 red pen for every 5 students. Set up a proportion to determine how many students the teacher has if he needs 7 red pens, being sure to match up units: . Cross-multiply to get x = 35 students. If the teacher uses 2 dry erase markers for each of the 35 students, he will use 2(35) = 70 dry erase markers. The correct answer is (C).

4.D

The question asks about the mean (average) of a set of numbers in a chart. Read the chart carefully to find the correct numbers, which are in the maximum elevation row. Use the formula T = AN, where T is the total, A is the average, and N is the number of things. The total is 239 + 289 + 329 + 240 + 195 = 1,292, and the number of things is 5. The equation becomes 1,292 = A(5). Divide both sides by 5 to get A = 258.4. The correct answer is (D).

5.A

The question asks about the number of defective widgets based on information about a random sample of the total produced. Since the widgets were randomly selected, the incidence of those defective in the sample should match that of the total produced. Start by ballparking. Less than half the widgets were defective, so the answer must be less than half of 5,913. Eliminate (D), which is too large. To extrapolate the sample results, set up a proportion. In this case, the proportion is based on the number of defective widgets out of the total of each group: . Cross-multiply to get 200x = 242,433. Divide both sides by 200 to get x = 1,212.165, or approximately 1,200. The correct answer is (A).

6.B

The question asks for the tax added to a bill based on a percentage. Translate the question in bite-sized pieces. Percent means “out of 100,” so 5% can be written as . When a percent is taken of a value, that percent is multiplied by the value, so this becomes . This is closest to $6.00. The correct answer is (B).

7.D

The question asks for measurements and gives conflicting units. Start by ballparking. The given measurements are in centimeters and millimeters, which are smaller than meters. Therefore, the number of meters for each measurement will be smaller than the number of centimeters and millimeters. Eliminate (A) and (B), which have very large measurements. Compare the remaining answers. The only difference is the third measurement. Start by converting the 5 millimeters to centimeters. The question states that 1 centimeter = 10 millimeters. Set up a proportion to determine how many centimeters are in 5 millimeters, being sure to match up units: . Cross-multiply to get 10x = 5. Divide both sides by 10 to get x = 0.5 centimeters. Next, convert 0.5 centimeters to meters. The question states that 1 meter = 100 centimeters. Set up a proportion to determine how many meters are in 0.5 centimeters, being sure to match up units: . Cross-multiply to get 100x = 0.5. Divide both sides by 100 to get x = 0.005 meters. As a fraction, this is . The correct answer is (D).

8.D

The question asks for a certain value on a graph. Height is listed along the horizontal axis, so find 11 on that axis. It will be between the lines for 8 and 12, closer to the 12. From this point, trace up to find the intersection with the line of best fit, using the answer sheet as a straight edge if necessary. It is between the horizontal gridlines for 30 and 40 on the vertical Growth rate axis. Only the value in (D) falls between 30 and 40. The correct answer is (D).

9.B

The question asks for the graph that best represents data in a table. Pick a piece of information from the table and use it to eliminate graphs in the answer choices. The graphs all look similar for small numbers of participants, so try a large number. If there are 20 participants, the price of the tour is $250. The numbers for Tour participants are listed along the horizontal axis in each answer, so find 20 on that axis. From this point, trace up to find the intersection with the graph, using the answer sheet as a straight edge if necessary. For (A), it is on the gridline for $300. This does not match the target value of $250, so eliminate (A). For (B), it is on the gridline for $250, so keep (B), but check the remaining answers just in case. For (C) and (D), there is no price indicated for 20 tour participants, so eliminate (C) and (D). The correct answer is (B).

10.A

The question asks for a value based on a ratio. The question states the ratio of almond flour to coconut flour is 7:38. To solve a ratio question, determine the number of parts in each group, then determine the number of groups in the whole. To do this, add the ratio numbers to get the number of parts: 7 + 38 = 45. To find the number of groups of 45 that are needed to get to the whole of 180, set up an equation: 45x = 180. Divide both sides by 45 to get x = 4. Use the ratio number for almond flour parts and the number of groups in the whole to calculate the actual volume of almond flour: 7(4) = 28 cups. The correct answer is (A).

11.B

The question asks for an accurate comparison based on a system of equations. Start by reading the full question, which gives equations for the number of cars passing through two intersections. Then label the parts of the equations with the information given. The question states that t is the total number of cars passing through the intersection and d is hours of daylight. Next, use Process of Elimination to get rid of answer choices that are not consistent with the labels. Choice (A) refers to the number of cars per hour increasing at a certain rate. To check this, plug in some numbers for d. Plug in d = 1, d = 2, and d = 3 to see what happens over time.

d = 1

d = 2

d = 3

For Intersection X: t = 450(1) = 450

450(2) = 900

450(3) = 1,350

For Intersection Y: t = 150(1) = 150

150(2) = 300

150(3) = 450

Each hour, the number of cars passing through Intersection X increases by 450, and the number of cars passing through Intersection Y increases by 150. Therefore, for either intersection, the rate is constant and not increasing at all for either intersection. Eliminate (A) and (C), since both refer to an increasing rate. Compare the remaining answers. The only difference is whether the number of cars passing through Intersection X or Intersection Y is greater. Choice (D) says that the number of cars passing through Intersection Y is greater, which is not true. Intersection X has greater numbers at all values of d, so eliminate (D). The correct answer is (B).

12.B

The question asks for an equation in terms of a specific variable. Although there are variables in the answer choices, plugging in on this question would be difficult given the number of variables. Instead, solve for n. To begin to isolate n, multiply both sides of the equation by n to get np = m. Divide both sides by p to get . The correct answer is (B).

13.B

The question asks for an inequality that models a specific situation. Translate the English to math in bite-sized pieces. The days are represented as d. The inequality must show the number of days before she hits or goes over her limit or 7 Gigabytes. This means that the expression containing d must be less than or equal to 7, which is represented as “≤ 7.” Eliminate (C) and (D), as the expression for days is not less than or equal to 7. The question asks about data use of 2 hours each day and 0.25 Gigabytes for each of those hours, so Maria uses 0.25(2) = 0.5 Gigabytes of data each day, represented as 0.5d. Eliminate (A), as it does not show 0.5 Gigabytes used each day. The correct answer is (B).

14.C

The question asks for the purchase price of the cell phone to the nearest dollar. Start by ballparking. The purchase price does not include the tax or the flat fee for the phone charger, so the purchase price must be less than the actual total paid. Eliminate (A), as it is not less than the total of $685.72 paid for the cell phone. Since the question asks for a specific value and the answers contain numbers in decreasing order, plug in the answers. Begin by labeling the answers as “purchase price” and start with (C), $625. The question states that the cell phone will have a 6.5% sales tax on the purchase price. “Percent” means divide by 100. Then multiply the result by the value of which the percent is taken. This becomes 0.065($625) = $40.625. Add this to $625 to get $665.63. The question also states that a flat fee of $20 is added for the phone charger, so this becomes $665.63 + $20 = $685.63. This nearly matches the value given in the question, which asks for the price to the nearest dollar, so stop here. The correct answer is (C).

15.B

The question asks for an estimate of the number of people in a group who would have rated a game at a certain level or higher based on an evaluation completed by a portion of the group. Since the people were randomly selected, the results found in the evaluation should match that of the larger group. To extrapolate the evaluation results, set up a proportion being sure to match up units. In this case, the proportion is based on the ratings of a video game given by members of the group. The total number of people that completed the survey was 14 + 29 + 13 + 18 = 74. Of those, 13 people rating the video game ✓+ and 18 people rating the video game ✓++, so the number of people who rated it with a ✓+ or higher was 13 + 18 = 31. The proportion becomes . Cross-multiply to get 74x = 7,440. Divide both sides by 74 to get x = 100.54. The question asks for an approximation, so round this to 100. The correct answer is (B).

16.C

The question asks for a new data point based on how it changes the mean and median of a set of data. The median is easier to determine, so start by finding the median of the existing data. The median of a list of numbers is the middle number when all values are arranged in order. The shoulder heights are not listed in order, so start by listing the shoulder heights from shortest to tallest. Elephant B has the shortest shoulder height of 1.83 meters. Elephant G has the 2nd shortest shoulder height of 1.90 meters, and Elephant A has the 3rd shortest shoulder height of 2.04 meters. Elephant C, Elephant E, Elephant F, and Elephant D have shoulder heights of 2.13, 2.45, 2.68, and 3.14 meters, respectively. Therefore, the median shoulder height of the original 7 elephants is 2.13 meters. The question states that Elephant H’s shoulder height increases the median value of the group of elephants. Eliminate (A) which is smaller than 2.13 meters and would decrease the median value. Eliminate (B) which matches the median and therefore would not increase it. The question also refers to a change in the mean or average value, so calculate the average shoulder height. Use the formula T = AN, where T is the total, A is the average, and N is the number of things. The total is 1.83 + 1.90 + 2.04 + 2.13 + 2.45 + 2.68 + 3.14 = 16.17, and the number of things is 7. The equation becomes 16.17 = A(7). Divide both sides by 7 to get A = 2.31. The question states that Elephant H’s shoulder height decreases the mean value of the group. Eliminate (D), as this value is equal to the mean and would keep the mean the same. The correct answer is (C).

17.D

The question asks for the total distance in meters covered by four animals over specified periods of time given the distance they can cover in kilometers in 10 minutes of running. Use Bite-Sized Pieces and Process of Elimination to tackle this question. The distance covered by the cheetah and the African wild dog can be read directly from the table, since the table’s data is based on an interval of 10 minutes. According to the table, the cheetah covers 18 km in 10 minutes, and the African wild dog covers 12 km in 10 minutes. Use proportions to determine the other two distances. Begin by setting up a proportion to calculate the distance covered by the lion in 20 minutes, being sure to match up units: . Cross-multiply to get 10x = 280, then divide both sides by 10 to get x = 28 km. Repeat the process to determine the distances for the zebra. The zebra ran for 40 minutes: . Cross-multiply to get 10x = 440, or x = 44 km. The total distance covered by the four animals in the specified time periods is 28 km + 18 km + 12 km + 44 km = 102 km. The question asks for an answer in meters, so set up a proportion to convert the units, being sure to match up units: . Cross-multiply to get x = 102,000 m. The correct answer is (D).

18.A

The question asks for the animal running for a specified period of time that covers a distance that is equivalent to the sum of two other animals’ distances over specified periods of time. Use Bite-Sized Pieces and Process of Elimination to tackle this question. Begin by calculating the distance covered by a cheetah running for 15 min and a giraffe running for 5 min. Set up a proportion to determine the distance covered by a cheetah in 15 minutes, being sure to match up units: . Cross-multiply to get 10x = 270, then divide both sides by 10 to get x = 27 km. Repeat the process to determine the distance covered by a giraffe in 5 minutes: , which becomes 10x = 45, or x = 4.5 km. The total distance covered by a cheetah and a giraffe in the specified time periods is 27 km + 4.5 km = 31.5 km. Use a proportion to calculate the distance covered by each animal in each answer choice, and eliminate answer choices that are not close to this value. Choice (A) becomes , then 10x = 330, or x = 33 km. Keep (A) for now, as it is very close to the target value of 31.5, but check the remaining answer choices to see if one is closer. Choice (B) becomes , then 10x = 240, or x = 24 km. Eliminate (B), as it is not closer to 31.5 km than (A). Choice (C) becomes , then 10x = 700 km, or x = 70 km. Eliminate (C). Choice (D) becomes , then 10x = 420 km, or x = 42 km. Eliminate (D). The correct answer is (A).

19.D

The question asks for a set of inequalities that are true for the slope and the y-intercept of a line on a graph. To find the correct set of inequalities, compare features of the graph to the answer choices. The equation is given in slope-intercept form, so r is the slope of the line and s is the y-intercept. The graph for this question has a y-intercept of 3 and a negative slope. Eliminate answer choices that do not match this information. Choices (A) and (B) say r, the slope, is positive. Eliminate (A) and (B), since they are not consistent with the graph’s negative slope. Choice (C) says s, the y-intercept, is less than 1. Eliminate (C) since this is not consistent with the graph’s y-intercept of 3. The correct answer is (D).

20.A

The question asks for the best interpretation of the slope of a line representing a set of data. In this question, y represents the height of a vine, and x represents the number of days since the vine was planted. The slope of a graph is defined as the change in y over the change in x, so the slope must relate to height as it increases over time. Eliminate (B) and (C), as they reference the height on specific days. Eliminate (D), as x itself represents days, so the slope cannot also be the number of days. Another way to approach this is to plug and play to see what gives a value that matches the value of the slope. Slope is calculated using the equation slope = . Calculate the slope of the line using two points, such as (1, 7.5) and (2, 9.0). The slope formula becomes . Choice (A) refers to the amount that the height of the vine is increasing each day. Each day, the vine’s height increases by 1.5 inches. Keep (A), but check the remaining answer choices just in case. Choice (B) refers to the height of the vine after the seventh day of growth. The vine grows 1.5 inches each day, so the vine will be 15.0 + 1.5 = 16.5 inches on the seventh day. Eliminate (B), as this does not match the value of 1.5 inches. Choice (C) refers to the height of the vine when it was first planted. The question states that the starter plant had a height of 6 inches. Since the vine’s height when planted is not 1.5 inches, eliminate (C). Choice (D) refers to the number of days the plant will keep growing, but no information is given about how long the plant will grow. Either way, the correct answer is (A).

21.C

The question asks for the total number of students with red hair in a school district based on a percentage. There are approximately 350 students per school in the 9 different schools, so there are approximately 9(350) = 3,150 total students in the district. The question states 1.5% of the district’s students have red hair. “Percent” means divide by 100. Then multiply the result by the value of which the percent is taken. This becomes 0.015(3,150) = 47.25 students have red hair in the district. The correct answer is (C).

22.53

The question asks for the total number of vouchers sold based on the cost of each voucher, the total voucher sales, and the total number of drinks sold. Translate the question in bite-sized pieces and write a system of equations for the situation. Use x to represent the number of vouchers sold for 1 drink and y to represent the number of vouchers sold for 2 drinks. One piece of information says that a voucher for 1 drink costs $6, so the sales for vouchers for 1 drink would be represented as 6x. The question states that a voucher for 2 drinks costs $10, so the sales for vouchers for 2 drinks would be represented as 10y. The question also states that the voucher sales total was $462, so 6x + 10y = 462. The question states that 89 drinks were purchased with vouchers. Each voucher represented as x is for 1 drink and each voucher represented as y is for 2 drinks, so the equation to represent the total number of drinks is x + 2y = 89. Solve the system of equations by stacking. Multiply the second equation by —6 and add it to the first equation to eliminate the x variable.

The result is —2y = —72. Divide both sides by —2 to get y = 36. Plug y back into either equation to solve for x. The second equation becomes x + 2(36) = 89, or x + 72 = 89. Subtract 72 from both sides to get x = 17. The total number of vouchers sold, or x + y, is 17 + 36 = 53. The correct answer is 53.

23. or 0.7

The question asks for the value of a variable in a system of equations. Solve the system of equations using stacking. When stacking the system of equations, multiply the second equation by 2 and add it to the first equation to eliminate the b variable.

The new equation becomes 5a = 4. Divide both sides by 5 to get a = 0.8. Plug a back into either equation to solve for b. The first equation becomes —3(0.8) + 6b = 1.8, or —2.4 + 6b = 1.8. Add 2.4 to both sides to get 6b = 4.2. Divide both sides by 6 to get b = 0.7. The correct answer is 0.7 or .

24.24

The question asks for the number of people ages 18—29 who participated in a survey. This question is much longer and more complicated than Q25, so start with that question, filling in the table with one piece of information at a time. The information determined in Q25 is bolded in the table below.

This question states that 25.6% of the people who chose coffee as their preferred beverage were 45—64 years old. The total number of people who prefer coffee is 43, so 0.256(43) = 11 people ages 45—64 prefer coffee. There were 12 people ages 45—64 who prefer tea, so 11 + 12 = 23 people ages 45—64 surveyed in total. These numbers are bolded in the table below.

The total number of people surveyed is 116, so 116 — 14 — 23 — 55 = 24 people ages 18—29 surveyed in total. The completed table is shown below.

The correct answer is 24.

25.43

The question asks for the total number of people surveyed who prefer coffee. Start by filling in the table one piece of information at a time. There were 55 people ages 30—44 surveyed and 21 of those prefer coffee, so 55 — 21 = 34 people ages 30—44 prefer tea. There were 14 people ages 65+ surveyed and 4 of those prefer coffee, so 14 — 4 = 10 people ages 65+ prefer tea. These numbers are bolded in the table below.

The total number of people who prefer tea is 17 + 34 + 12 + 10 = 73. The total people surveyed is 116, so 116 — 73 = 43 total people prefer coffee. These numbers are bolded in the table below.

The correct answer is 43.